Вы находитесь на странице: 1из 11

20052 + 2 x 2005 + 1995 + 19952 800

(2005 + 1995)2 = 4002 = 20 00.



800 800 0

Singapore Mathematical Society Singapore Mathematical Olympiad 2005

(Senior Section Solutions)

1. Ans: C

Since 2005 and 2007 are odd numbers, so are 20052007 and 20072005. Thus, 20052007 + 20072005 is even, and it is divisible by the smallest prime number 2.

2. Ans: A

3. Ans: D

Since the equation 2y2 - 8y - p = 0 has only one solution, its discriminant

( _8)2 - 4 x 2 x (-p) = O.

Thus p = -8.

4. Ans: B

One easily checks that for n 2 10, n! will end with two zeros. Also,

I! + 2! + 3! + 4! + 5! + 6! + 7! + 8! + 9!

= 1 + 2 + 6 + 24 + 120 + 720 + 5040 + 40320 + 362880 = 409113.

Thus the sum of the last two digits is 1 +3=4.

5. Ans: B

5002200510g50022005 = 500iogs00220052005 = 20052005.

6. Ans: A

Since RS = OP, we have RS = OR. Hence, LROS = LRSO = 12°. Then

LOQR = LORQ = 2 x 12° = 24°.

Thus,

LPOQ = 180° - LQOR - LROS

= 180° - (180° - 24° - 24°) - 12° = 36°.

17

1 16 108 «16 108

- (- + - + xy) ~ - x - x xy = 12

3 x y x Y

16 108

====} - + - + xy ~ 36.

x y

7. Ans: E

Using Arithmetic Mean ~ Geometric Mean, we have

Equality is achieved when 16 108 = xy = 36 = 12, which is satisfied when

x y 3

x = ~ and y = 9. Thus the smallest possible value is 36.

8. Ans: E

We simply replace x by -y and y by -x to get

2(-y) -1

-x = ====} xy + x = -2y - 1

-y-1

-l-x

====} y = .

2+x

9. Ans: B

V 4x4 + x8 - 2x4 + 1

2(1 + 4)

4x

2X2 + x4 + 1

2( 2X2 )

(x2 + 1)2

x2

x2 + 1

since x > O.

x

10. Ans: C

x2 + y2 _ 2x + 2y = 2 {=::::} (x2 - 2x + 1) + (y2 + 2y + 1) = 4 {=::::} (x - 1)2 + (y + 1)2 = 22.

Therefore, the points (x, y) in the Cartesian plane satisfying the equation consist of points on the circle centred at (1, -1) and of radius 2. For a point (x, y) on the circle, it is easy to see that its distance V x2 + y2 from the origin (0, 0) is greatest

18

when it is the point in the fourth quadrant lying on the line passing through (0,0) and (1, -1). In other words,

(x,y) = (1 + V2, -1- V2), and x2 + y2 = (1 + \1'2)2 + (-1- \1'2)2 = 6 + 4\1'2.

11. Ans: 193

(2 + V3)4 = [(2 + V3)2]2 = (4 + 4V3 + 3)2

= (7 + 4V3)2 = 49 + 56V3 + 48 ~ 193.9.

Thus, the greatest integer less than (2 + /3)4 is 193.

12. Ans: 60

By Pythagoras' Theorem, BC = V1692 - 1562 = 65. Let the perpendicular from C to AB meet AB at D. Then 6ABC r-;» 6AC D. Thus,

CD BC x 65

- = - ====} - = - ====} x = 60.

AC AB 156 169

13. Ans: 7

Let a = 3x - 27 and b = 5x - 625, so that the equation becomes

a 2 + b2 = (a + b) 2 ====} 2ab = ° ====} a = ° or b = 0.

When a = 0, we have 3X - 27 = ° ====} x = 3. When b = 0, we have 5x - 625 = ° ====} x = 4. Hence, the sum is 3 + 4 = 7.

14. Ans: 12

Let x be the smallest integer. Then we have

x3 S 2808 S x(x + 6).

The first inequality implies that x S 14, while the second inequality implies that x ~ 11. If x = 13, then the product of the other integers is 23 X 33 = 216. This is impossible, since the smallest factor of 216 greater than 13 is 18 but 182 = 324 > 216. Therefore, we must have x = 12. The other two integers are easily seen to be 13 and 18.

15. Ans: 1

Let t = 2004. Then the expression is equal to

19

F

16. Ans: 6

1 1 y'3 3x 3x + y'3 1

f(x)+ f(1-x) = 3x + y'3+ 31-x + y'3 = 3xy'3 + 3 + 3 + 3xy'3 = 3 + 3xy'3 = y'3'

Therefore,

V3[j( -5) + f(6)] + V3[j( -4) + f(5)] + V3[j( -3) + f(4)] + V3[f( -2) + f(3)] + V3[f( -1) + f(2)] + V3[j(0) + f(I)]

(1 1 1 1 1 1)

= V3 y'3 + y'3 + y'3 + y'3 + y'3 + y'3 = 6.

17. Ans: 2049

Without loss of generality, we may assume that A ::; B. It follows that 1000 ::; A ::; 1024, and a direct check shows that 1024 is the only number between 1000 and 1024 that divides 41 x 52 x 210. Thus, A = 1024 and B = 1025. Therefore,

A + B = 1024 + 1025 = 2049.

18. Ans: 192

Let AB = e ern, BC = a cm and C A = b cm. By the cosine rule,

Let 0 be the centre of the circle. Then LBOC = 2LA = 120°. Moreover, OB = OC = 4 cm. Therefore, by the cosine rule,

a2 = 42 +42 - 2 x 4 x 4 cos 120° = 48 ==? a = 4V3.

Then be = a2 - 4 = 48 - 16 = 32. Thus,

1 1 y'3

area (D.ABC) = "2besin 60° = "2 x 32 x :2 = 8V3.

Therefore, x = 8y'3 and x2 = (8y'3)2 = 192.

19. Ans: 4

e2 = ab - 16 = (8 - b)b - 16 = _b2 + 8b - 16

==? e2 + b2 - 8b + 16 = 0 ==? e2 + (b - 4)2 = 0 ==? e = 0 and b = 4.

20

Since b = 4, it follows that a = 8 - 4 = 4. Thus, a + c = 4 + 0 = 4. In this solution, we have used the fact that the sum of two non-negative numbers is zero only when both numbers are zero.

20. Ans: 8 Observe that

So the only possiblility is that a; = 1 for each i = 1,2, ... ,8. Hence al + .. ·+as = 8.

21. Ans: 143

Observe that for any positive number x,

1

Vx+l-yIX ~

-yx--t-1- x

(Vx+l)2 - (yIX)2 - + yx.

Thus we have

111

---=:-----= + + ... + -------===

V4+V5 V5+v6 vn+vn+l

= (Vs - V4) + (v6 - Vs) + ... + (vn+l - yin)

=vn+l-01.

Hence, vn+l - V4 = 10 ==? vn+l = 12 ==? n + 1 = 144 ==? n = 143.

22. Ans: 211

1 1

---

A B

192 B - A 192

20052 _ 20042 -¢==} A x B = 4009 -¢==} 4009 x (B - A) = 192 x A x

The prime factorization of 4009 is given by 4009 = 19 x 211. Since 192 = 26 x 3 and B > A, it follows that A = 19 and B = 211.

23. Ans: 108

Let the area of DAGF be SI and the area of DAGE be S2. Let the area of DAlvlG be S, so that the area of DABM is also S, since M is the midpoint of BG. Now,

AG·AF S2 AG·AE

and -

AM·AG S AM·AB·

Therefore,

S2 AG . AE . A1VI . AG 3 3

-= =-x2=-

SI AM·AB·AG·AF 4 2'

~ GE x 3 x

Note that -S = -G = -. Therefore, - = - ==? x = 108.

1 F 72 2 72

21

x6 - 2V3x5 - X4 + X3 - 4x2 + 2x - V3

= X4(X2 - 2V3x - 1) + X(X2 - 4x + 1) + x - V3 = 0 + 0 + 2 + V3 - V3 = 2.

24. Ans: 2

x = 1 = 1 . 2 + V3 = 2 + V3 = 2 + V3.

2-V3 2-V3 2+V3 4-3

Then x - 2 = V3 ==? x - V3 = 2 ==? x2 - 4x + 1 = 0 and x2 - 2V3x - 1 = o.

Thus,

22

25. Ans: 90

Since a, b are the roots of the equation x2 - (c + 4)x + 4( c + 2) = 0, it follows that

a + b = c + 4 and ab = 4(c + 2).

Then a2 + b2 = (a + b)2 - 2ab = (c + 4)2 - 8(c + 2) ==? a2 + b2 = c2. Hence the triangle is right-angled, and x = 90.

26. Ans: 25

Observe that ~(a + b + la - bl) = max(a, b). Also, x2 > x - 1 since x2 - x + 1 = (x - ~)2 + ~ > O. Thus, we have

Thus the equation becomes x2 = 35x - 250 ==? (x - 25)(x-1O) = 0 ==? x = 25, 10. Therefore, the largest value of x is 25.

27. Ans: 36

To determine the number of partitions, we just have to decide the number of ways of inserting a partition between two consecutive vowels if we insert a partition at all. Thus the total number of partitions is 4 x 3 x 3 = 36.

28. Ans: 9011

1 1 1

-- = -- + 2004 = -- + 2003 + 2004

a2005 a2004 a2003

1 2004 x 2005

= - + (1 + 2 + 3 + ... + 2004) = 1 + = 2009011.

a1 2

Hence, _1_ - 2000000 = 9011.

a2005

29. Ans: 401

By comparing the coefficients of xk, it is easy to see that ak = kL Letting x = 1, we get

ao + a 1 + ... + a k = (1 + 1) (1 + 2) (1 + 3) ... (1 + k) = (k + I)!

Therefore, ao + al + ... + ak-l = (k + I)! - k! = k· k!. Note that 2005 = 5 x 401, where 401 is a prime number. It is easy to see that a prime number p divides k! only if k 2:: p. Therefore, the smallest possible value of k is 401.

30. Ans: 1

Since the exponent on the left side of the given equation is non-zero if x > 0, it follows that the equation is possible only when

Since x > 0, it follows that x = 1.

31. Ans: 6

We can rearrange the equation to get

x2 - x(2 + y) + y2 - 2y = O.

As a quadratic equation in the variable x, the discriminant of the above equation is given by

Discriminant = (2 + y)2 - 4(1)(y2 - 2y) = 4 + 12y - 3y2 = 16 - 3(y - 2)2.

Since x is an integer, the discriminant must be a perfect square and thus it is non-negative. Hence,

16 4

16 - 3(y - 2)2 2:: 0 ===? (y - 2)2 ~ :3 ===? Iy - 21 < v'3 < 3.

Since y is an integer, it follows that its only possible values are given by y = 0, 1,2,3,4. When y = 1,3, the discrimant is 12, which is not a perfect square. Thus, y = 0,2,4.

When y = 0, we have x2 - 2x = 0 ===? .T = 0,2. When y = 2, we have x2 - 4x = 0 ===? x = 0,4. When y = 4, we have x2 - 6x + 8 = 0 ===? x = 2,4.

Thus, there are 6 solutions, namely (0,0), (2,0), (0,2), (4,2), (2,4), (4,4).

32. Ans: 133

From (i), it is easy to see that the whole ordered 7-tuple is determined by al and a2. It is easy to check that a6 = 3al + 5a2· Thus by (ii), we have 3al + 5a2 = 2005. It follows that al is divisible by 5. Write al = 5k. Then we have 15k + 5a2 = 2005 ===? 3k + a2 = 401. The possible values of a2 are {2, 5, 8, ... ,398}, if both k

23

and a2 are positive. Note that each of the above values of a2 determines a unique positive integral value of al satisfying the equation 3al + 5a2 = 2005. Therefore, there are 133 such sequences.

33. Ans: 12

Let 0: be an integer satisfying the given equation, so that

40:2 - (4V3 + 4)0: + V3n - 24 = 0 ===} 40:2 40: - 24 = V3(40: - n).

Since v'3 is irrational, it follows that

40:2 - 40: - 24 = 0 and 40: - n = O.

Substituting 0: = ~ into the first equation, we have

4(~)2 - 4(~) - 24 = 0

===} n2 4n - 96 = 0 ===} (n - 12)(n + 8) = 0 ===} n = 12, -8.

Since n is a positive integer, we have n = 12.

34. Ans: 2

Subtracting the second equation from the first one, we have

xy + yz = 255 - 224 = 31 ===} y(x + z) = 31.

Since 31 is a prime number, we have y = 1 and x + z = 31. Together with the first given equation, we have

x . 1 + x(31 - x) = 255 ===} (x - 15) (x - 17) = 0

===} x = 15 or 17.

When x = 15, y = 1, we have z = 16. When x = 17, y = 1, we have z = 14.

Therefore, there are two such solutions, namely (15,1,16) and (17,1,14).

24

35. Ans: 210

Let abed = 1000a+ 100b+ 10e+d be a four-digit number divisible by 7. In particular, 1 :::; a :::; 9. We are given that dbea = lOOOd + 100b + 10e + a is also divisible by 7. Observe that

abed - dbea = 999a - 999d = 999(a - d).

Since 7 does not divide 999, it follows that 7 must divide a-d. Thus a == d mod 7. Since 7 divides 1001, we have

1000a + 100b + 10e + d == 0 mod 7 ==? - a + 100b + 10e + d == 0 mod 7 ==? 100b + 10e == 0 mod 7

==? 10(10b + c) == 0 mod 7

==? lOb + e == 0 mod 7.

Therefore we must have a = d mod 7 and lOb + e == 0 mod 7.

Conversely, by reversing the above arguments, one easily sees that if a == d mod 7 and lOb + e == 0 mod 7, then both abed and dbea are divisible by 7.

N ow there are 14 pairs of (a, b) satisfying a == d mod 7, which are

(1,1), (2,2), (3,3), ... , (9,9), (1,8), (2,9), (7,0), (8, 1), (9,2).

Since 0 :::; lOb + e :::; 99, there are 15 pairs of (b, c) satisfying lOb + e == 0 mod 7 (including (0,0)). In fact, this is equal to the number of non-negative integers less that 100 that are divisible by 7. Therefore, there are 14 x 15 = 210 numbers with the required properties.

25

-

Singapore Mathematical Society Singapore Mathematical Olympiad (SMO) 2005

(Senior Section, Special Round Solutions)

1. Since the sums of the digits of the two numbers are the same, they leave the same remainder when they are divided by 9. Thus their difference is divisible by 9. So the possible answers are 36 or 81. It is easy to see that both cases are possible: For example: (645,564) and (218,182).

2. Let M and N be points on AB and AD so that MIll BC and N J II DC. Let a = D I = C F, and b = B J = C E. Then

D.KJB ~ D.K1M, D.NJE ~ D.DCE,

D.LJ N ~ D.L1 D D.BCF ~ D.M1F

Thus

KJ

b

NJ

b+CJ

KJ+J1 M1 a+1C

b

J1 +1L NJ b+CJ

1L

a

M1

a+1C

a

Multiplying the four equations we get

(K J) ( J 1) + (K J) (I L ) (I L ) ( J I) + (K J) (I L )

ab ab

which yields KJ = 1L.

A

M

N

3. Suppose that this result is not true. Observe that S has total of C20) = 45 2-element subsets. Let Si = {XiI, Xi2}, i = 1,2, ... ,45, be the 45 2-element subsets of Sand s, = XiI + Xi2· Then by the assumption, the 45 values s; are mutually distinct. Since 3 :::; Si :::; 47, and there are exactly 45 numbers from 3 through 47, we have

26

{s, : i = 1, ... ,45} = {3, 4, ... ,47}. Since there is pair summing to 3 and a pair summing to 47, the numbers 1,2,23,24 E S. But then 1 + 24 = 2 + 23 gives rise to a contradiction.

4. Let m = 1000100000000. Let Mn = (m + n)!, where n is an integer such that o :s; ri :s; 10000. Observe that if Mn = abede ... , where a, b, e, d, ... are the digits of Mn, then the first 4 digits of Mn+I are (i) abed, in which case the fifth digit is a + e > e, (ii) the first four digits of abed + 1 or (iii) the first four digits of abed + 2. The last case can happen only when a = 9. Therefore we see that if a < 9, among the numbers Mo, MI, ... , MlO, there is at least one for which the first four digits are the first four digits of xyzw + 1 where xyzw are the first four digits of Mo. It follows that the first four digits of the numbers numbers Mo, MI, ... , MlOOOO includes all of 1000,1001, ... ,8999. Hence the answer is yes.

27

-

Вам также может понравиться